The price of tiling a room varies directly as the size of the room. Sam is laying tile in his kitchen. If the tiling costs $4,224.00 for 264 square feet, what is the size of a kitchen that costs $3,824.00?

The Price Of Tiling A Room Varies Directly As The Size Of The Room. Sam Is Laying Tile In His Kitchen.

Answers

Answer 1

Answer:

239 ft².

Step-by-step explanation:

Let P represent the price for tiling.

Let S represent the size of the room.

From the question,

Price (P) varies directly as the size (S) i.e

P & S

P = KS

Where K is the constant of proportionality.

Next, we shall determine the value of K as follow

Price (P) = $ 4224

Size (S) = 264 ft²

Constant of proportionality (K) =?

P = KS

4224 = K × 264

Divide both side by 264

K = 4224/264

K = 16

Finally, we shall determine the size of the kitchen that will cost $ 3824 for tiling.

This is illustrated below:

Price (P) = $ 3824

Constant of proportionality (K) = 16

Size (S) =?

P = KS

3824 = 16 × S

Divide both side by 16

S = 3824/16

S = 239 ft²

Therefore, the size of the kitchen is 239 ft².


Related Questions

What is the y−intercept of the line that passes through the point (4,9)and is parallel to the line y=12x+2?

Answers

Answer:

y- intercept = - 39

Step-by-step explanation:

The equation of a line in slope- intercept form is

y = mx + c ( m is the slope and c the y- intercept )

y = 12x + 2 ← is in slope- intercept form

with slope m = 12

Parallel lines have equal slopes, thus

y = mx + c ← is the partial equation

To find c substitute (4, 9) into the partial equation

9 = 48 + c ⇒ c = 9 - 48 = - 39 ← y- intercept

Answer:

y-intercept = -39

Step-by-step explanation:

if two lines are parallel it means they have the same gradient so we compare the equation given to the default equation of a line

y=mx+c

y=12x+2

comparing we have the gradient m=12 now finding the equation of the line parallel to the given line we use

y-y1=m(x-x1)

y1=9 and x1=4

y-9=12(x-4)

y-9=12x-48

y=2x-48+9

y=2x-39

comparing to the default equation of a line y=mx+c where c is the y-intercept

therefore the y-intercept is -39

Gene is playing a game with a bag of marbles. 3 of the marbles are blue, 4 are green, and 7 are yellow. See below for awarded prizes. $2 green $0.5 yellow $4 blue What is the expected cost (or payout for Gene's game?

Answers

Answer:

$23.5

Step-by-step explanation:

Gene is playing a game with a bag of marbles. If 3 of the marbles are blue, 4 are green, and 7 are yellow and awarded prices for the marbles are $2 green $0.5 yellow $4 blue, the expected payout for Gens game is expressed as shown;

If a blue marble costs $4, 3 blue marbles will cost 3*$4 = $12

If a green marble costs $2, 4 green marbles will cost 4*$2 = $8.0

If a yellow marble costs $0.5, 7 yellow marbles will cost 7*$0.5 = $3.5

Total payout for Gene's game will be the equivalent to $12+ $8 + $3.5 = $23.5.

Hence Gene expected cost will be $21.5

Wait times at a dentist's office are typically 21 minutes, with a standard deviation of 2 minutes. What percentage of people should be seen by the doctor between 17 and 25 minutes for this to be considered a normal distribution?

Answers

ANSWER: 95%

HOW:
95% of a group of data with a normal distribution is between two standard deviations to the right and left

Answer:

95%

Step by step explanation:

z = 17-21 / 2 and z = 25-21/2

z=-2 (2.28%) z=2 (97.72%)

97.72 - 2.28 = 5.44

100% - 5.44% is about equal to 95%

Show all work to solve the equation for x. If a solution is extraneous, be sure to identify it in your final answer.

square root of the quantity x minus 3 end quantity plus 5 equals x

Answers

Answer:

Step-by-step explanation:

[tex]\sqrt{x-3} +5=x\\\sqrt{x-3} =x-5\\squaring ~both~sides\\x-3=x^2-10x+25\\x^2-10x-x+25+3=0\\x^2-11x+28=0\\x^2-7x-4x+28=0\\x(x-7)-4(x-7)=0\\(x-7)(x-4)=0\\x=7,4[/tex]

put x=7 in the given equation

[tex]\sqrt{7-3} +5=7\\\sqrt{4} +5=7\\2+5=7\\7=7[/tex]

which is true .

∴ x=7 is a solution of the given eq.

now put x=4 in the given eq.

[tex]\sqrt{4-3} +5=7\\1+5=7\\6=7\\[/tex]

which is not true.

∴x=4 is an extraneous solution.

a second degree equation in one variable example how many solutions does it have ?a second degree equation in one variable example how many solutions does it have ? is it possible to have many solutions or no solutions tions give an example for each

Answers

Answer:

  0, 1, or 2 real solutions

Step-by-step explanation:

Including complex and repeated solutions, a polynomial with real coefficients, and of degree n, always has n solutions.

If you're only concerned about real solutions, a 2nd degree equation in one variable may have 0, 1, or 2 real solutions. Here are some examples.

0 solutions: x^2 +1 = 01 solution: x^2 = 02 solutions: x^2 -1 = 0

what is the answer for 6x-4=-26+5x

Answers

Answer: x=-22

Step-by-step explanation:

    6x-4=-26+5x

6x-4-5x=-26+5x-5x ⇔ subtraction property of equality

       x-4=-26

  x-4+4=-26+4 ⇔ addition property of equality

         x=-22

Answer:

x = - 22

Step-by-step explanation:

6x - 4 = - 26 + 5x

First of all group like terms

Send the constants to the right side of the equation and those with variables to the left

That's

6x - 5x = 4 - 26

Simplify

We have the final answer as

x = - 22

Hope this helps you

Complete the table. At least the first few so I understand how to do it

Answers

Answer:

What we need to do is simply multiply the values in both columns e.g 4 * 3/36 = 12/36

Please check explanation for complete answer

Step-by-step explanation:

Here, we are concerned about filling the empty columns of the table.

What we want to do here is simply straightforward. All we need to do is to

multiply the values of x by the values of P(x) in each of the individual rows.

Also recall, we do not need to reduce the fractions.

So we have;

2. 3 * 2/36 = 6/36

3. 4 * 3/36 = 12/36

4. 5 * 4/36 = 20/36

5. 6 * 5/36 = 30/36

6. 7 * 6/36 = 42/36

7. 8 * 5/36 = 40/36

8. 9 * 4/36 = 36/36

9. 10 * 3/36 = 30/36

10. 11 * 2/36 = 22/36

11. 12 * 1/36 = 12/36

Which of the following symbols could correctly finish the statement. Select all that apply. 0___-8 = ≠ > < ≥ ≤

Answers

Answer:

>

Step-by-step explanation:

Even though its 0 its still greater than any negative number.

Answer:

Step-by-step explanation:

simplify 3/7
into a whole number

Answers

Answer:

You can't because it's not a "full fraction" like 7/7. The best you can do is turn that into a decimal like 0.43

Step-by-step explanation:

help8b2 • 2b3 a) 16b-2 ,b) 16b6 ,c) 16b-4 ,d) 16b5

Answers

Answer:

16b^5

Step-by-step explanation:

8b^2×2b^3

= 16b^2+3

= 16b^5

Answer:

D)16b5

Step-by-step explanation:

8b2×2b3

= 16b2+3

= 16b5

Please help me understand this. Thank you! Gina has borrowed 100 songs from her friend. She plans to download an equal number of songs on her music player each week for 5 weeks. The graph shows the number of songs left to download, y, for a certain number of weeks, x: A graph titled Song Downloading shows the Number of Weeks on x-axis and Number of Songs Left to Download on the y-axis. The x-axis scale is shown from 0 to 5 at increments of 1, and the y-axis scale is shown from 0 to 140 at increments of 20. A straight line joins the ordered pairs 0, 100 and 1, 80 and 2, 60 and 3, 40 and 4, 20 and 5, 0. Part A: What is the rate of change and initial value of the function represented by the graph, and what do they represent in this scenario? Show your work to find the rate of change and initial value. (6 points) Part B: Write an equation in slope-intercept form to model the relationship between x and y. (4 points)

Answers

Answer:

Rate of Change/Slope = -20

Equation: y= -20x +100

Step-by-step explanation:

A. We know the rate of change is also known as the slope.  If we used the slope formula to find the slope we can find the Rate of Change.

[tex]\frac{y2 - y1}{x2 - x1} = \frac{100-80}{1-2} = \frac{20}{-1} = -20[/tex]

B. Since we know the slope and 1 point on the graph we can substitute them in for 'b'

(0,100)

(100)=-20(0) + b

b = 100

Since we know the slope and the b value we can write the equation:\

y = -20x +100

The rate of change refers to the slope of the line, that is the change in y-axis per unit change in the value on the x-axis. Hence, the rate of change is :

Slope = - 20y = -20x + 100

Slope = Rise / Run

Rise = (y2 - y1) = (0 - 100) = - 100

Run = (x2 - x1) = (5 - 0) = 5

Slope = - 100 / 5 = - 20

General form of a slope - intercept relation :

y = bx + c

The intercept, c can be calculated thus:

100 = - 20(0) + c

100 = 0 + c

c = 100

Hence, the slope - intercept equation will be y = - 20x + 100

Learn more : https://brainly.com/question/18479471

Please answer this question now

Answers

Answer:

30.9 cm²

Step-by-step explanation:

To find the surface area of this figure, we find the area of the base and the 3 identical sides.

The base is split into two identical right triangles. Let's find the area of one and multiply by two.

Half of 3: 1.5

[tex]1.5\cdot2.6=3.9\\3.9\div2=1.95[/tex]

There are two right triangles:

[tex]1.95\cdot2=3.9[/tex]

The area of one of the sides will be the same thing, except the height is 6.

[tex]1.5\cdot6=9\\9\div2=4.5\\4.5\cdot2=9[/tex]

There are 3 sides identical to this one:

[tex]9\cdot3=27[/tex].

Add 27 and 3.9:

[tex]27+3.9=30.9[/tex]

Hope this helped!

Answer:

30.9 square centimeters

Step-by-step explanation:

3 * 1/2(3)(6) + 1/2(3)(2.6) = 30.9

If the geometric mean of a and 120 is 60, find the value of a.

Answers

Answer:

The answer is option 4.

Step-by-step explanation:

Given that the mean formula is total score/number of score. So we can assume that total score is (a+120), number of score is 2 and the mean value is 60. Then you have to find the value of a :

[tex] \frac{a + 120}{2} = 60[/tex]

[tex]a + 120 = 60 \times 2[/tex]

[tex]a + 120 = 120[/tex]

[tex]a = 120 - 120[/tex]

[tex]a = 0[/tex]

[tex] \text{ Geometric mean of two numbers } a \text{ and } b , \text{ where } a,b>0 \text{ is given by } G= \sqrt{ab}[/tex]

[tex] \sqrt{a\cdot120}=60 [/tex]

[tex] \implies 60 \cdot 2 \cdot a=60\cdot 60 \qquad \text{Squaring both sides} [/tex]

[tex] \implies a =30 [/tex]

HELP ASAP ITS SO HARD! Kelsey did the following division problem. Her teacher says that the quotient she found is wrong. −2 5/6 ÷ 1 1/3 −17/6 ÷ 4/3 −6/17• 3/4 −6×3 divided by 17×4 −18/68 −9/34 A. Identify what Kelsey did wrong in her calculations. B. Find the correct quotient, showing all of your calculations.

Answers

Part A

Her steps were

[tex]-2 \frac{5}{6} \div 1 \frac{1}{3}\\\\-\frac{17}{6} \div \frac{4}{3}\\\\-\frac{6}{17} \times \frac{3}{4}\\\\-\frac{6\times 3}{17\times4}\\\\-\frac{18}{68}\\\\-\frac{9}{34}\\\\[/tex]

Kelsey made a mistake on line 3. Note how the 17/6 flips to 6/17. This is not correct. You keep the first fraction the same, but you do flip the second fraction. This only applies when you divide two fractions.

The third step should look like [tex]-\frac{17}{6}\times \frac{3}{4}[/tex]

=======================================================

Part B

Here's what she should have written

[tex]-2 \frac{5}{6} \div 1 \frac{1}{3}\\\\-\frac{17}{6} \div \frac{4}{3}\\\\-\frac{17}{6} \times \frac{3}{4}\\\\-\frac{17\times 3}{6\times 4}\\\\-\frac{51}{24}\\\\-\frac{17}{8}\\\\[/tex]

If you want to convert that improper fraction to a mixed number, then you could do something like this

[tex]-\frac{17}{8} = -\frac{16+1}{8}\\\\-\frac{17}{8} = -\frac{16}{8}-\frac{1}{8}\\\\-\frac{17}{8} = -2 \frac{1}{8}\\\\[/tex]

Or you could divide 17 over 8 using long division to get 2 remainder 1. The 2 is the quotient that goes to the left of the 1/8. The remainder of 1 is the numerator of 1/8.

**Yoxelt buys 4 1/2 gallons of soda. One-fourth of the soda he bought was Pepsi and the rest was Sprite. How many gallons of Pepsi did Yoxelt buy? Show all work below.

Answers

Answer:

  1 1/8

Step-by-step explanation:

1/4 of the 4 1/2 gallons were Pepsi, so the amount is ...

  (1/4)(9/2) = (1·9)/(4·2) = 9/8 = 1 1/8

Yoxelt bought 1 1/8 gallons of Pepsi.

Which data set matches the box-and-whisker plot?
A) 12 13 15 19 23 23 25 26.5 28 30
B) 15 13 19 21 23 24 27 29 32
C) 11 31 13 15 19 21 21 25 27 29 31
D) 11 13 15 19 23 23 24 26.5 28 33​

Answers

Answer:

D) 11 13 15 19 23 23 24 26.5 28 33​

Step-by-step explanation:

The box-and-whisker plot displayed above has the following key values that we can use to identify which of the given data set it matches. It has:

Minimum value = 11

Q1 = 15

Median = 23

Q3 = 26

Maximum value = 33

From the options given, using just the max and min value, we can conclude that the data set in option D matches the box plot.

The data set in option D has a minimum value of 11, and a maximum value of 33.

in a 10 team league, each teams play every other team exactly twice. find the total number of games played in the league

Answers

Since it’s a 10 team league and each team plays with the other team twice then 10+10=20, thus there would be 20 games played

Addition can be defined as the process of adding two numbers. The total number of games played in the league is 90.

What is Addition?

Addition can be defined as the process of adding two numbers such that the result is the combined value of the two numbers.

Given that in a 10-team league, each team play every other team exactly twice. Therefore, the total number of games that will be played by the first team is 18, similarly, the number of games that will be played by the second team is 16.

Therefore, as mentioned above the series will continue with a common difference of -2 and will continue till 0. Thus, the total number of games played in the league is,

Total number of games = 18 + 16 + 14 + 12 + 10 + 8 + 6 + 4 + 2 = 90

Learn more about Addition:

https://brainly.com/question/13167637

#SPJ2

What is the difference between sin^-1 and sin?

Answers

Answer:

Step-by-step explanation:

sin of angle x is the trig ratio sine of x.

sin-1 x is the angle whose sine is x.

sin-1 x can also be written as arcsin x.

Which of the following are natural numbers? There may be more than one correct answer. Select all that apply. If only one answer is correct, select "only" and the answer that applies. A.) only B.) −1,−2,−3,… C.) 7,8,9,… D.) fractions E.) 22

Answers

Answer:

Option C and option E

Step-by-step explanation:

C.) 7,8,9,…

E.) 22

Natural numbers are also called counting numbers.

They begin from 1, 2, 3 to infinity.

Natural numbers are greater than zero (0)

They do not have decimal point in them.

They are positive integers, as such they do not have minus

Natural numbers can include commas when they are large like 3,000

Two students use different methods to solve this multiplication problem:
3/4*-4 2/9
Read each of their methods below and then enter numbers to correctly complete their
work.

Answers

Answer/Step-by-step Explanation:

Given, [tex]\frac{3}{4}*-4\frac{2}{9}[/tex]

Ivy solves this problem by writing each number as a fraction and then multiplies as shown below:

[tex] \frac{3}{4}*-4\frac{2}{9} = \frac{3}{4}* -\frac{38}{9} [/tex]

[tex] = \frac{3}{4}*-\frac{38}{9} = -\frac{3*38}{4*9} = -\frac{1*19}{2*3}[/tex]

[tex]= -\frac{19}{6}[/tex]

Fabian solves this problem by writing the mixed number as a sum and applies the distributive method of multiplication as shown below:

[tex] \frac{3}{4}*-4\frac{2}{9} = \frac{3}{4}(-4 + -\frac{2}{9}) [/tex]

[tex] = \frac{3}{4}*-4 + \frac{3}{4}*-\frac{2}{9} [/tex]

[tex] = -\frac{3}{1} + \frac{1}{2}*-\frac{1}{3} [/tex]

[tex] = -3 + (-\frac{1}{6}) [/tex]

[tex] = -3 - \frac{1}{6} = -3\frac{1}{6} [/tex]

please helppp ill give brainliest the question is attached below

Answers

Answer: It is false, the diameter is not 16 m.

Step-by-step explanation:

P 4m are 8 m are 32 m away. The diameter= 32 m.

(I hope its rights!!!)

Carey earns $9.75 working part time on weekends. The table below shows the amount, a, Carey earns for working h hours. Carey’s Earnings h 0 1 3 a $0 $9.75 ? Which value completes the table to show the amount Carey earns for working 3 hours?

Answers

Answer:

$29.25

Step-by-step explanation:

For every 1 hour, Carey earns $9.75.  Multiply $9.75 by 3 to find out how much she earns for 3 hours of work.

$9.75 × 3 = $29.25

Carey earns $29.25 for working 3 hours.

Answer:

29.25

Step-by-step explanation:

I got it right on edge!! trust me

what equation accurately represent this statement three less than 4 times a number is less than 12

Answers

Answer: 4t - 3 < 12

Step-by-step explanation:

WILL GIVE BRAINLIEST

Question 8(Multiple Choice Worth 1 points) (06.04 LC) Choose the correct product of (6x − 2)(6x + 2). A.36x2 + 4 B.36x2 − 24x + 4 C. 36x2 + 24x + 4 D. 36x2 − 4

Answers

Answer:

D) 36x²-4

Step-by-step explanation:

36x²+12x-12x-4

36x²-4

[tex](6x - 2)(6x + 2) \\ = {(6x)}^{2} - {(2)}^{2} \\ = {36x}^{2} - 4[/tex]

Answer:

D.

[tex] {36x}^{2} - 4[/tex]

Hope you could understand.

If you have any query, feel free to ask.

help help help me plZZZZZ ill give you brainly ;DDD

Answers

Answer:

the answer is 60.7

Step-by-step explanation:

60 to has a between numbers like given in the picture

so as number line it's

60.1 . 60.2 60.3 60.4 60.5 60.6 60.7 60.8 and continue

if u get any 3 digit number like 600 to 650 in number line

u do it like it the same 600.1 600.2.... and go on

Answer:

63½ or 63.5

Step-by-step explanation:

65-60=5

10points=5

1point=?

1×5/10= ½

that means the sequence continues after adding ½ i.e

60..60½...61...61½...62...62½...63...63½...64..64½...65

you have been asked the 8th number which is 63½

help plz I think the first one is correct but I'm not sure

Answers

Answer: You are correct.

The left side 10x+25 is the cost expression for Black Diamond, while the right hand side 5x+50 is for Bunny Hill. The x is the number of hours.

Find f(x) and g(x) so the function can be expressed as y = f(g(x)). (1 point) [tex]y=\frac{7}{x^{2} } +10[/tex]

Answers

Answer:

The functions are [tex]f(x) = 7\cdot x+10[/tex] and [tex]g(x) = \frac{1}{x^{2}}[/tex], respectively.

Step-by-step explanation:

Let suppose that [tex]g(x) = \frac{1}{x^{2}}[/tex], then [tex]f(g(x))[/tex] is:

[tex]f(g(x)) = 7\cdot \left(\frac{1}{x^{2}} \right) + 10[/tex]

[tex]f(g(x)) = 7\cdot g(x) + 10[/tex]

Thus,

[tex]f(x) = 7\cdot x + 10[/tex]

The functions are [tex]f(x) = 7\cdot x+10[/tex] and [tex]g(x) = \frac{1}{x^{2}}[/tex], respectively.

ASAP PLZ ANSWER!!! Can you tell me step by step to this question 8,595 ÷ 24?

Answers

Answer:

358 and remainder of 3

Step-by-step explanation:

1. Divide it like any other problem

24 goes into 85, 3 times with 13 left overBring down the 9 and 24 goes into 139, 5 times with 19 left overThen bring down the 5 and 24 goes inside 195, 8 times with 3 left overSo your remainder would be 3

Hope this helps

-7p+2(5p-8)=6(p+6)-7

Answers

Answer:

-15

Step-by-step explanation:

-7p+10p-16=6p+36-7

3p-16=6p+29

3p-6p=29+16

-3p=45

p=45/-3

p=-15

P=5

-7p+2(5p-8)=6(p+6)-7
Distribute
-7p+10p-16=6p+36-7
Add -7p and 10p together
3p-16=6p+36-7
Subtract 7 from 36
3p-16=6p+29
+6p (on both sides)
9p-16=29
+16 (on both sides)
9p=45
Divide both sides by 9
P=5

What is m Round the answer to the nearest whole number.
O 30°
O 35°
O 55°
O 60°

Answers

Answer:

30

Step-by-step explanation:

fufyfuf7fjcjcufuy7fufucyyxyvkbuvufudydy shut up

Other Questions
Ahmad has some files.He gaveof the files and had 14 files left.5How many files did he have at first? X = y + 12How to solve for variable Why do turtles hv paddle like flippers ? Evaluate. Write in standard form. Solid cesium bromide has the same kind of crystal structure as CsCl which is pictured below: If the edge length of the unit cell is 428.7 pm, what is the density of CsBr in g/cm3. The sum of two numbers is 15. One number is 101 less than the other. Find the numbers. BRAINLIEST FOR BEST ANSWER!!!! Explain the causes and consequences of Presidents Johnsons impeachment. Which number is in the 3rd position after ordering indescending order. V220,-10, V100, 11.5 On March 15, a fire destroyed Sheridan Company's entire retail inventory. The inventory on hand as of January 1 totaled $5900000. From January 1 through the time of the fire, the company made purchases of $2032000, incurred freight-in of $242000, and had sales of $4140000. Assuming the rate of gross profit to selling price is 20%, what is the approximate value of the inventory that was destroyed In a frequency distribution of 290 scores, the mean is 99 and the median is 86. One would expect this distribution to be: All of the following statements about MapReduce are true EXCEPT: MapReduce handles the complexities of network communication. MapReduce is a general-purpose execution engine. MapReduce handles parallel programming. MapReduce runs without fault tolerance. v divided by 5 is equal to 60. George wanted a home in the St. Paul area. He was willing to pay premium price for it even though there were other homes in other areas that were better values. When George was asked why he was willing to pay so much, he told the sales associate that he worked in that area and that he wanted his children to attend school in a specific school district. George's willingness to pay top dollar for a home in St. Paul is because of which characteristic of value? What does the root jur mean Thanks for the help :)Write a short paragraph on Hinduism, describe its origin, major principles, and how it affected existing laws, social practices, or culture. Also, describe whether and how the religion influenced other societies. i need help really bad Why is it necessary to separate oxgynated blood and deoxgynated blood in living organisums? Two football teams, the Raiders and the 49ers are engaged in a tug-of-war. The Raiders are pulling with a force of 5000N. Which of the following is an accurate statement? A. The tension in the rope depends on whether or not the teams are in equilibrium. B. The 49ers are pulling with a force of more than 5000N because of course theyd be winning. C. The 49ers are pulling with a force of 5000N. D. The tension in the rope is 10,000N. E. None of these statements are true. A box is filled with 8 blue cards, 6 red cards, and 6 yellow cards. A card is chosen at a random from the box. What is the probability that the card is not red ? Write your answer as a fraction. Click an item in the list or group of pictures at the bottom of the problem and, holding the button down, drag it into thecorrect position in the answer box. Release your mouse button when the item is place. If you change your mind, dragthe item to the trashcan. Click the trashcan to clear all your answers.Type the expressions as radicalsy3/5